identité avec la fonction d'Euler

Bonsoir,

Il y a peu, dans un autre sujet, Ben a rappelé la banque d'exercices proposée par Michel Quercia sur son site. Parmi ceux-ci, il est proposé l'identité suivante : pour tout entier $n \geqslant 2$, démontrer que $$\sum_{k=1, \, \mbox {pgcd}(k,n) = 1}^{n} k = \frac {n \varphi(n)}{2}.$$

Les outils traditionnels de théorie analytique des nombres permettent de régler cette question en 2-3 lignes. Ainsi, il m'a paru intéressant de voir comment les différents intervenants, avec leurs connaissances et leurs idées propres, résolvent ce type de problème.

A vos crayons et bon courage,

Borde.

Réponses

  • Bonsoir Borde

    En regardant dans le groupe cyclique d'ordre $n$, les $k$ que l'on somme sont les éléments primitifs, il y en a $\varphi(n)$.
    Or pour $n \neq 2$, si $k$ est primitif, $n-k$ l'est aussi et est différent de $k$, leur somme fait $k+(n-k)=n$ et il y a $\dfrac{\varphi(n)}{2}$ tels couples. Donc au total $n\cdot \dfrac{\varphi(n)}{2}$
    Le cas $n=2$ se vérifie directement immédiatement.
    Alain
  • bonjour à tous,

    1)On a aussi : $\varphi (n^2)= n \varphi (n)$,
    il est donc équivalent de montrer que :$2 \sum ..=\varphi(n^2)$,
    mais je ne vois pas trop comment conclure avec ceci.

    2)Hors question initiale , mais concerne$\varphi$:
    lu dans Merveilleux Nombres Premiers de JP Delahaye:
    On a démontré que :
    $\frac {n\sqrt{3}}{\sqrt{\varphi(1) + \varphi(2)+.... +\varphi(n)}}$ a pour limite le nombre $\pi$.
    ( sans référence ; question: est-ce techniquement délicat ?, où trouver une preuve ?)
    Merci

    Bonne journée.
  • Bravo Alain,

    La démarche algébrique fonctionne très bien ici (mais c'est un maître en la matière qui a répondu...).

    Pour bs,

    L'ordre moyen de la fonction $\varphi$ est dans mon livre, exo 4.14 page 130, qui montre que, si $x \rightarrow \infty$, on a $$\frac{1}{x} \sum{n \leqslant x} \varphi(n) \sim \frac{x}{2 \zeta(2)},$$ ce qui te donne ton résultat. En fait, l'exo 4.14 donne plus, puisque tu as une estimation du terme d'erreur, que l'on n'a pas dans le Delahaye.

    A propos, le terme d'erreur que l'on trouve dans mon livre n'est pas le meilleur : en 1963, utilisant les derniers raffinements de Vinogradov (qui sont toujours actuels), Walfisz, dans son livre posthume, a montré que : $$\sum{n \leqslant x} \varphi(n) = \frac{x^2}{2 \zeta(2)} + O \left ( x (\ln x)^{2/3} (\ln \ln x)^{4/3} \right ),$$ alors que je n'ai proposé seulement que l'estimation : $$\sum{n \leqslant x} \varphi(n) = \frac{x^2}{2 \zeta(2)} + O \left ( x \ln x \right ).$$ En 1998, le suisse Pétermann a englobé ce résultat dans un théorème plus général, reprenant les mêmes idées que Walfisz, mais utilisant des estimations de sommes d'exponentielles plus précises. Pour ceux qu'un article technique n'effraie pas, lire \lien {http://almira.math.u-bordeaux1.fr/jtnb/1998-1/jtnb10-1.html#jourelec} (cliquer sur le dernier article. Attention : il est en postscript). Comme toujours, je reste, ainsi que les spécialistes de TAN du forum sans doute (Brux, Bob, Gaston,...) à la disposition de celle ou celui qui souhaiterait avoir des explications complémentaires.

    Comme toujours en TAN, le passage de "mon" terme d'erreur à celui de Walfsifz est très très très délicat, et nous fait passer du niveau L3/M1 à celui de la recherche.

    Ce soir, je donnerai ma version de l'identité ci-dessus, si, d'ici là, personne d'autre ne l'a fait.

    Bien sûr, d'autres idées peuvent émerger...

    Borde.
  • Bravo Alain,

    La démarche algébrique fonctionne très bien ici (mais c'est un maître en la matière qui a répondu...).

    Pour bs,

    L'ordre moyen de la fonction $\varphi$ est dans mon livre, exo 4.14 page 130, qui montre que, si $x \rightarrow \infty$, on a $$\frac{1}{x} \sum_{n \leqslant x} \varphi(n) \sim \frac{x}{2 \zeta(2)},$$ ce qui te donne ton résultat. En fait, l'exo 4.14 donne plus, puisque tu as une estimation du terme d'erreur, que l'on n'a pas dans le Delahaye.

    A propos, le terme d'erreur que l'on trouve dans mon livre n'est pas le meilleur : en 1963, utilisant les derniers raffinements de Vinogradov (qui sont toujours actuels), Walfisz, dans son livre posthume, a montré que : $$\sum_{n \leqslant x} \varphi(n) = \frac{x^2}{2 \zeta(2)} + O \left ( x (\ln x)^{2/3} (\ln \ln x)^{4/3} \right ),$$ alors que je n'ai proposé seulement que l'estimation : $$\sum_{n \leqslant x} \varphi(n) = \frac{x^2}{2 \zeta(2)} + O \left ( x \ln x \right ).$$ En 1998, le suisse Pétermann a englobé ce résultat dans un théorème plus général, reprenant les mêmes idées que Walfisz, mais utilisant des estimations de sommes d'exponentielles plus précises. Pour ceux qu'un article technique n'effraie pas, lire \lien {http://almira.math.u-bordeaux1.fr/jtnb/1998-1/jtnb10-1.html#jourelec} (cliquer sur le dernier article. Attention : il est en postscript). Comme toujours, je reste, ainsi que les spécialistes de TAN du forum sans doute (Brux, Bob, Gaston,...) à la disposition de celle ou celui qui souhaiterait avoir des explications complémentaires.

    Comme toujours en TAN, le passage de "mon" terme d'erreur à celui de Walfsifz est très très très délicat, et nous fait passer du niveau L3/M1 à celui de la recherche.

    Ce soir, je donnerai ma version de l'identité ci-dessus, si, d'ici là, personne d'autre ne l'a fait.

    Bien sûr, d'autres idées peuvent émerger...

    Borde (message précédent à virer. Merci. Désolé).
  • Bonsoir,

    Borde,grand merci pour le lien entre Delahaye et ton livre; je ne suis pas encore parvenu à la page 130, mais certainement que je n'aurais pas fait le rapprochement.Les deux livres sont maintenant annotés.

    En attendant l'approche analytique à ta question : trois remarques concernant $\varphi$, pour tous.

    1) question:$\varphi$ est clairement injective, mais est-elle surjective ?

    2)info lue sur Delahaye : deux conjectures de Carmichaël:
    2.1 :n est premier $ n = 1 [mod :\varphi(n)]$
    2.2 :pour tout n>0, il existe au moins un nombre $m \neq n$ tel que $\varphi(n)=\varphi(m)$

    3) c'est la dernière question de l'exercice 3.154 du Aleph d'algèbre Terminale CE:
    Soit $n=p^aq^br^c$, $a \geq b \geq c$, p, q, r, premiers distincts;démontrer:
    $\varphi(n) \leq {n-a}$;
    j'ai l'impression que c'est vrai pour tout n, mais sans parvenir à le démontrer.

    Merci
  • Pour n impair, $\varphi(n)=\varphi(2n)$

    Donc l'injectivité me paraît douteuse. :)
  • pardon, non injective,merci, et pourtant j'ai relu !
  • Pour la surjectivité c'est douteux aussi il me semble que phi est toujours pair pour n>2.
  • Exact, puisque $\varphi(n)$ est pair dès que $n \geqslant 3$.

    Puisque j'ai repris la main, voici une autre preuve de l'identité proposée en début de ce fil, preuve plus dans l'esprit "théorie multiplicative des nombres", dans le sens où l'on somme des fonctions multiplicatives.

    Le principe est toujours le même : lorsque l'on somme sur un certain ensemble d'entiers, on cherche d'abord une identité donnant la fonction indicatrice de cet ensemble.

    Cela peut parfois être facile (sommer sur des multiples d'un nombre premier, par exemple), ou bien cela peut parfois être extrêmement compliqué (sommer sur les progressions arithmétiques a conduit Dirichlet, vers 1837, a inventer la rihe notion de "caractères").

    Ici, on utilise une vieille identité de la fonction de Möbius : $\displaystyle {\sum_{d \mid n, \, d \mid m} \mu(d) = 1}$ si $\mbox {pgcd}(m,n) = 1$ et $0$ sinon, ce qui fournit immédiatement la fonction indicatrice cherchée.

    On note $S_n$ la somme cherchée. En injectant l'identité précédente, et en intervertissant les sommations, il vient : $$S_n = \sum_{d \mid n} \mu(d) \sum_{m \leqslant n/d} md = \sum_{d \mid n} d \mu(d) \frac {1}{2} \times \frac {n}{d} \left ( \frac {n}{d} + 1 \right ) = \frac {n^2}{2} \sum_{d \mid n} \frac {\mu(d)}{d} + \frac {n}{2} \sum_{d \mid n} \mu(d),$$ et on conclut avec : $$\sum_{d \mid n} \frac {\mu(d)}{d} = \frac {\varphi(n)}{n},$$ et $$\sum_{d \mid n} \mu(d) = 0,$$ si $n \geqslant 2$, et rappelons que cette dernière somme vaut $1$ si $n=1$.

    Ainsi, on a obtenu : $S_1 = 1$ et $\displaystyle {S_n = \frac {n \varphi(n)}{2}}$ si $n \geqslant 2$.

    Borde.
  • Lire "riche" au lieu de "rihe" au milieu du message précédent.

    Borde.
  • Bonjour,
    merci , oui, bien sûr; décidément... je reformule la question relative à la "surjectivité":
    pour tout q pair >=2, existe-t-il n tel que $\varphi(n)=q$ ?
    Bonne journée
  • Super! gros mercis à Guimauve et à Borde.
  • Juste une remarque pour Alain Debreil: (je peux me tromper)

    $k\mapsto n-k$ est une involution de $(\Z/n\Z)^*$ donc on peut écrire

    \[S=\sum_{k\wedge n=1}k=\sum_{k\wedge n=1}(n-k)=n\varphi(n)-S\]

    si je ne m'abuse... A-t-on vraiment besoin de traiter séparément le cas $n=2$?
  • Bonsoir Skilveg

    Ta démo est très élégante, et n'a effectivement pas le cas particulier $n=2$.

    Si $n=2$ l'involution est l'identité or dans mon raisonnement, il fallait que $k$ et $n-k$ soient distincts pour qu'ils apparaissent tout deux dans la somme. Il était donc nécessaire de traiter le cas $n=2$ séparément puisqu'alors la somme ne porte que sur un unique terme.

    Alain
  • J'ai oublié (impardonnable...), mais je remercie mille fois les contributeurs de ce fil, en particulier Alain, bs, Skilveg, Guimauve.

    Borde.
  • Bonjour Borde !
    J'ai une solution par récurrence à votre exercice sur l'identité avec l'indicatrice d'Euler :
    D'abord, on vérifie le résultat pour $n=2$.
    Ensuite, je tiens à m'excuser à l'avance pour la manière dont je vais écrire, car je ne sais pas encore me servir de Latex (mais je vais apprendre avec un ami à moi qui a déjà quelques bases).
    J'utiliserai donc les notations Maple. [J'ai fait la traduction en \Latex :) AD]

    Soit $n\geq 2$ un entier.
    Le principe est simple, au départ : au lieu de calculer $\displaystyle \sum\limits_{\mathrm{pgcd}(k,n)=1} k$ je calcule $\displaystyle \sum\limits_{\mathrm{pgcd}(k,n)=1} \dfrac{k}{n}$.

    Mon idée est en fait de regarder les fractions $\frac 1n , \frac 2 n , \frac 3 n , \ldots , \frac n n $, chacune ayant été simplifiée en $\frac k d$, où $k$ et $d$ sont premiers entre eux, et $d$ étant bien-sûr un diviseur de $n$.
    J'écris que : $$ \sum_{\substack{1\leq k\leq n \\ \mathrm{pgcd}(k,n)=1}} \frac k n = \sum_{1\leq k \leq n} \frac k n \ \ - \text{"ce qui reste"}.
    $$ Et ce qui reste, c'est justement toutes les fractions que j'ai pu réduire, ie : $$ \displaystyle \sum\limits_{\substack{d\mid n\\d \neq n}} \sum\limits_{\mathrm{pgcd}(k,d)=1} \dfrac{k}{d}. $$ Attention, il faut enlever $d=n$ car $\dfrac 1 n$ doit être compté dans la somme à calculer.

    Or justement, par récurrence, (une récurrence forte),
    $\displaystyle \sum\limits_{\mathrm{pgcd}(k,d)=1} \dfrac{k}{d} $ vaut ou bien $\dfrac{\varphi(d)}{2}$ lorsque $d>1$, ou bien 1 si $d=1$.
    Donc la quantité vaut : $\displaystyle \sum\limits_{\substack{d\mid n\\1 \neq d \neq n}} \dfrac{\varphi(d)}{2} + 1$, soit : $\displaystyle \frac n 2 - \frac{\varphi(n)}{2} - \frac{\varphi(1)}{2} +1$ car $\displaystyle \sum\limits_{d\mid n} \varphi(d) = n$.
    Or $\varphi(1)=1$, donc finalement la somme recherchée vaut : $$ \dfrac{n+1}{2} - \left(\dfrac n 2 - \dfrac{\varphi(n)}{2} + \dfrac 1 2 \right) = \dfrac{\varphi(n)}{2}$$ et on a ainsi le résultat en multipliant par $n$ (la somme que j'ai calculée est celle des $\dfrac k n$ ).
  • Bonjour Borde !
    J'ai une solution par récurrence à votre exercice sur l'identité avec l'indicatrice d'Euler :
    D'abord, on vérifie le résultat pour $n=2$.
    Ensuite, je tiens à m'excuser à l'avance pour la manière dont je vais écrire, car je ne sais pas encore me servir de Latex (mais je vais apprendre avec un ami à moi qui a déjà quelques bases).
    J'utiliserai donc les notations Maple. [J'ai fait la traduction en \Latex :) AD]

    Soit $n\geq 2$ un entier.
    Le principe est simple, au départ : au lieu de calculer $\displaystyle \sum\limits_{\mathrm{pgcd}(k,n)=1} k$ je calcule $\displaystyle \sum\limits_{\mathrm{pgcd}(k,n)=1} \dfrac{k}{n}$.

    Mon idée est en fait de regarder les fractions $\frac 1n , \frac 2 n , \frac 3 n , \ldots , \frac n n $, chacune ayant été simplifiée en $\frac k d$, où $k$ et $d$ sont premiers entre eux, et $d$ étant bien-sûr un diviseur de $n$.
    J'écris que : $$ \sum_{\substack{1\leq k\leq n \\ \mathrm{pgcd}(k,n)=1}} \frac k n = \sum_{1\leq k \leq n} \frac k n \ \ - \text{"ce qui reste"}.
    $$ Et ce qui reste, c'est justement toutes les fractions que j'ai pu réduire, ie : $$ \displaystyle \sum\limits_{\substack{d\mid n\\d \neq n}} \sum\limits_{\mathrm{pgcd}(k,d)=1} \dfrac{k}{d}. $$ Attention, il faut enlever $d=n$ car $\dfrac 1 n$ doit être compté dans la somme à calculer.

    Or justement, par récurrence, (une récurrence forte),
    $\displaystyle \sum\limits_{\mathrm{pgcd}(k,d)=1} \dfrac{k}{d} $ vaut ou bien $\dfrac{\varphi(d)}{2}$ lorsque $d>1$, ou bien 1 si $d=1$.
    Donc la quantité vaut : $\displaystyle \sum\limits_{\substack{d\mid n\\1 \neq d \neq n}} \dfrac{\varphi(d)}{2} + 1$, soit : $\displaystyle \frac n 2 - \frac{\varphi(n)}{2} - \frac{\varphi(1)}{2} +1$ car $\displaystyle \sum\limits_{d\mid n} \varphi(d) = n$.
    Or $\varphi(1)=1$, donc finalement la somme recherchée vaut : $$ \dfrac{n+1}{2} - \left(\dfrac n 2 - \dfrac{\varphi(n)}{2} + \dfrac 1 2 \right) = \dfrac{\varphi(n)}{2}$$ et on a ainsi le résultat en multipliant par $n$ (la somme que j'ai calculée est celle des $\dfrac k n$ ).
  • Merci beaucoup d'avoir converti ma réponse en Latex! Je viens de la relire, j'ai dit une imprécision à la ligne 18 : j'enlève les termes correspondant à d=n car c'est justement lui que je cherche à calculer, et j'obtiendrais zéro sinon...

    J'avais une autre idée, au départ, pour démontrer l'identité :
    pour n premier, c'est ok, pour n de la forme p^k avec p premier, ce n'est pas très compliqué, car on connaît très facilement les nombres non premiers avec p^k, et j'essaie à présent de résoudre l'exercice pour n=a*b avec a et b premiers entre eux. Quelqu'un a-t-il une idée pour ce cas-là?
  • Bonsoir Victor-Emmanuel,

    Je regarderai ta preuve lorsque j'aurai plus de temps. A première vue, cela fonctionne...

    Borde.
  • Bonsoir, je relance le message por essayer d'avoir une autre démo de la jolie formule de Borde, partant peu-être sur ma dernière piste (cas où $n=ab$, avec $a$ et $b$ premiers entre eux.
Connectez-vous ou Inscrivez-vous pour répondre.